Domanda:
Perché lo stato $ S_ {z} = 0 $ è vietato ai fotoni?
Todd R
2012-12-12 10:32:07 UTC
view on stackexchange narkive permalink

Se i fotoni sono bosoni di spin-1, la meccanica quantistica non implica che i valori consentiti per la componente z dello spin (in unità di $ \ hbar $) siano -1, 0 e 1?

Perché allora in pratica usiamo solo gli stati $ \ pm 1 $?

Mi è stato detto che questo è direttamente correlato alle due polarizzazioni del fotone. Questo sembra essere più un argomento classico, tuttavia, derivante dal fatto che le equazioni di Maxwell non consentono onde EM longitudinali nel vuoto.

Ho anche sentito che è correlato al fatto che i fotoni non hanno massa a riposo, anche se capisco molto meno di questo ragionamento.

Quello che sto cercando sono elaborazioni su questi due argomenti (se li ho corretti), e forse un argomento su come questi due sono equivalenti (se esiste).

http://physics.stackexchange.com/q/73911/. Per il fotone usiamo l'elicità, non lo spin, perché le rappresentazioni senza massa del gruppo di Poincaré non possono essere caratterizzate dallo spin.
correlati: http://physics.stackexchange.com/q/29766/
Si noti che il contrario di questo è molto più facile da dimostrare: se c'è un vincolo su $ s_z $, la particella deve essere priva di massa. Questo perché se la particella fosse massiccia, potremmo entrare nel suo frame di riposo, e in quel frame non ci sarebbe alcun asse preferito da utilizzare per definire il vincolo sullo spin.
Recentemente sto leggendo il volume I del libro di Weinberg e mi piacciono le sue spiegazioni.Si risponde a questa domanda in modo esplicito o implicito "ma esattamente", da qualche parte?@acuri
Otto risposte:
ACuriousMind
2016-07-01 15:07:09 UTC
view on stackexchange narkive permalink

Le particelle prive di massa con spin non hanno uno stato "$ S_z = 0 $" perché in realtà non hanno spin come le particelle massicce . Hanno elicità , che è il valore della proiezione dell'operatore di rotazione sull'operatore di quantità di moto. La ragione di ciò è la teoria della rappresentazione del gruppo di simmetria spaziotemporale, il gruppo di Poincaré.

Per capirlo, dobbiamo prima ricordare che "spin" è il numero che etichetta le rappresentazioni irriducibili di $ \ mathrm {SU} (2) $, la doppia copertina del gruppo di rotazione $ \ mathrm {SO} (3) $. Ma, nella teoria quantistica relativistica dei campi, che è la teoria necessaria per descrivere i fotoni, questo gruppo di rotazione non è il gruppo di simmetria spaziotemporale che dobbiamo rappresentare. Invece, dobbiamo cercare rappresentazioni della componente connessa all'identità del gruppo di Poincaré $ \ mathrm {SO} (1,3) \ rtimes \ mathbb {R} ^ 4 $, cioè della corretta trasformazione ortocrona di Lorentz insieme alle traduzioni.

Ora, per le rappresentazioni a dimensione finita del gruppo di Lorentz, siamo fortunati in quanto esiste un'equivalenza "accidentale" delle rappresentazioni algebriche di $ \ mathfrak {so} (1,3) $ e $ \ mathfrak { su} (2) \ times \ mathfrak {su} (2) $, permettendoci di etichettare le rappresentazioni a dimensione finita in cui i campi relativistici classici si trasformano per coppie di mezzi interi $ (s_1, s_2) $ dove $ s_i \ in \ frac {1} {2} \ mathbb {Z} $ etichetta una singola rappresentazione $ \ mathfrak {su} (2) $. L'effettiva algebra di rotazione si trova diagonalmente in questo $ \ mathfrak {su} (2) \ times \ mathfrak {su} (2) $, quindi la rotazione fisica di tale rappresentazione è $ s_1 + s_2 $. Questo determina la rotazione classica associata a un campo .

Come spesso, la teoria quantistica rende le cose più complicate: il teorema di Wigner implica che dobbiamo ora cercare rappresentazioni unitarie del gruppo di Poincaré nel nostro spazio di stati di Hilbert. Ad eccezione della rappresentazione banale corrispondente al vuoto, nessuna delle rappresentazioni di dimensione finita è unitaria (essenzialmente perché il gruppo di Poincaré è non compatto e non ha sottogruppi normali compatti). Quindi dobbiamo rivolgerci a rappresentazioni a dimensione infinita, e qui non abbiamo l'equivalenza tra $ \ mathfrak {so} (1,3) $ e $ \ mathfrak {su} (2) \ times \ mathfrak {su } (2) $ . Le tecniche sfruttate per realizzare questa equivalenza si basano esplicitamente sulla dimensionalità finita della rappresentazione. In particolare , non esiste un isomorfismo come $ \ mathrm {SO} (1,3) \ cong \ mathrm {SU} (2) \ times \ mathrm {SU} (2) $, indipendentemente di quanto spesso leggerai affermazioni simili nei libri di fisica. Per ulteriori informazioni su questo problema, vedere ad es. questa risposta di Qmechanic.

Si scopre che classificare le rappresentazioni unitarie non è un compito così semplice. La classificazione completa è chiamata classificazione di Wigner, e risulta che per costruire rappresentazioni unitarie irriducibili, è rilevante guardare il piccolo gruppo corrispondente alla quantità di moto di una particella - il sottogruppo del gruppo di Lorentz che lascia invariante la quantità di moto della particella. Per una particella massiccia, questo è $ \ mathrm {SO} (3) $, e risulta che possiamo etichettare la rappresentazione unitaria anche con il nostro spin familiare $ s $.

Ma per una particella priva di massa, la quantità di moto $ (p, -p, 0,0) $ non è invariante sotto $ \ mathrm {SO} (3) $, ma sotto un gruppo chiamato $ \ mathrm {ISO} ( 2) $ o $ \ mathrm {SE} (2) $, che è essenzialmente $ \ mathrm {SO} (2) $ con le traduzioni. Essendo abeliano, $ \ mathrm {SO} (2) $ ha solo rappresentazioni unidimensionali irriducibili, etichettate da un unico numero $ h $, che risulta fisicamente essere l'autovalore dell'elicità. Ci sono casi più generali per $ \ mathrm {ISO} (2) $, chiamati rappresentazioni di rotazione continua (CSR), ma finora non sono stati fisicamente rilevanti.

Ora, questo singolo numero $ h $ capovolge il suo segno sotto la parità, quindi per particelle associate a campi classici con spin diverso da zero, dobbiamo prendere entrambe le rappresentazioni $ h $ e $ -h $. E questo è tutto: le particelle senza massa di elicità $ h $ hanno la rappresentazione $ h \ oplus -h $ nel loro spazio di stati, non una rappresentazione di spin di $ \ mathrm {SO} (3) $. La valutazione dell'operatore di spin effettivo mostra che la nostra idea classica di spin coincide con il numero $ h $.

Pertanto, senza aver detto nulla sul fotone o sul campo elettromagnetico in particolare, sappiamo che le particelle prive di massa di spin diverso da zero hanno due gradi di libertà . Questo è completamente generale e al centro dell'argomento che tutti i bosoni vettoriali senza massa sono bosoni di gauge :

Sappiamo che un campo vettoriale generico ha tre d.o.f. - le componenti di campo indipendenti che si trasformano l'una nell'altra durante la trasformazione di Lorentz, quindi tre insiemi indipendenti di operatori di creazione e annichilazione che si trasformano l'uno nell'altro, quindi ci aspettiamo tre tipi distinti di stati delle particelle.

Ma i due d.o.f. di una particella di spin-1 senza massa non corrisponde a questo - quindi uno dei d.o.f. di un campo vettoriale senza massa deve essere "falso". Il modo in cui i campi d.o.f.s sono "falsi" è che il campo è un campo di gauge e ci sono 1 d.o.f. nella libertà di scegliere un calibro. La storia della quantizzazione della teoria di gauge - anche nel caso abeliano dell'elettromagnetismo - è sottile, e hai ragione a non accettare ciecamente l'argomento secondo cui le due polarizzazioni classiche del campo di gauge - quella longitudinale è eliminata dalla simmetria di gauge - diventano tipi distinti di stati particellari nella teoria quantistica: il disaccoppiamento degli stati che si assocerebbero ingenuamente ai modi longitudinali è assicurato dalle identità di Ward e non è affatto ovvio a priori.

È per questo che le proprietà di essere un bosone di gauge e di non avere $ S_z = 0 $ e di essere senza massa sono tutte correlate: essendo una di queste cose immediatamente forza anche le altre due. In questa risposta, ho considerato "essere senza massa" come la proprietà fondamentale, poiché questo mostra "no $ S_z = 0 $" senza assumere nulla di più specifico sul campo - in particolare, senza limitarsi a misurare i campi o l'elettromagnetismo a priori.

Puoi commentare come questo si collega all'assorbimento dei fotoni?Puoi fare in modo che un atomo in uno stato $ S_z = 0 $ assorba un fotone e passi a uno stato $ S_z = ± 1 $, quindi lo "spin" del fotone si accoppia ai gradi di libertà del momento angolare meccanico.Come appare questo processo in termini di rappresentazioni di gruppo nella tua risposta?
@EmilioPisanty: Gli stati legati come gli atomi sono terribilmente difficili da trattare nella teoria quantistica dei campi, quindi penso che di solito si modifichi questo nel regime QM non relativistico e si imponga semplicemente che $ S_z = 0 $ non esiste per un fotone a mano.Puoi, tuttavia, fare in modo che un elettrone in QFT assorba o emetta un fotone in presenza di un nucleo - è bremsstrahlung (inverso), ma lì non hai $ S_z $ livelli per l'elettrone, l'elicità intrinseca del fotoneva solo nel momento angolare dell'elettrone.
Può essere difficile, ma rimane interessante.Nel solito MQ non relativistico, si pensa che l'assorbimento di fotoni passi da uno stato di $ J_z = + 1 $ globale ben definito a un altro stato simile.Qui dici che lo stato del fotone non è in quella rappresentazione, quindi cosa sta succedendo lì?
@EmilioPisanty: Con "orribilmente difficile" intendo dire che penso che sia un problema irrisolto.Le QFT interagenti hanno spazi di stati genericamente sconosciuti: nessuno sa come scrivere lo stato di un atomo, quindi non possiamo davvero chiederci come appare l'interazione in QFT.Non sono sicuro di quale sia il modo migliore per farlo - questo probabilmente costituisce di per sé una nuova domanda interessante.
Questo è molto carino, ma se l'obiettivo di questa domanda era fornire una comprensione intuitiva a persone che non hanno familiarità con QFT (un compito arduo, per essere sicuri), dubito che avrà successo.
"Ora, questo singolo numero h ribalta il suo segno sotto la parità" c'è qualche possibilità che potresti approfondire su questo?Come vediamo che la parità agisce in questo modo?(O dovrei chiederlo come domanda separata?
John Rennie
2012-12-12 14:10:00 UTC
view on stackexchange narkive permalink

Non posso migliorare la risposta di KDN, ma visti i commenti di Todd questo è un tentativo di riformulare la risposta di KDN in termini semplici.

Un sistema è solo in un autostato di rotazione attorno a un asse se una rotazione circa l'asse non cambia il sistema. Prendi $ z $ come direzione di viaggio, quindi per un sistema di rotazione 1 lo stato $ S_z $ = 0 sarebbe simmetrico a una rotazione attorno a un asse normale alla direzione di viaggio. Ma questo può essere il caso solo se la quantità di moto è zero, cioè nel frame di riposo. Se il sistema ha una quantità di moto diversa da zero, qualsiasi rotazione cambierà la direzione della quantità di moto in modo da non lasciare il sistema invariato.

Per una particella massiccia possiamo sempre trovare un frame di riposo, ma per un particella priva di massa non esiste un frame di riposo e quindi è impossibile trovare un'autofunzione di spin attorno a qualsiasi asse diverso dalla direzione di marcia. Questo vale per tutte le particelle prive di massa, ad es. i gravitoni hanno anche solo due stati di rotazione.

Questo è stato molto utile. Ero stanco di ascoltare la spiegazione "classica" delle polarizzazioni risultanti dalle equazioni di Maxwell, e non ho ancora familiarità con la teoria dei campi per comprendere appieno gran parte della matematica negli altri post (sebbene come esposizione alle idee e alla notazione della teoria dei campi sono molto utili di per sé). Questo argomento basato sulla simmetria e sulle idee di base dell'invarianza è stato molto utile e mi fornisce le basi per comprendere alcuni dettagli tecnici di altri post. Grazie @John Rennie e tutti gli altri che hanno pubblicato!
Questa è una bella ripresa con una spiegazione concettuale minima, ma penso che la semplificazione vada un po 'troppo oltre e diventi errata. In particolare, questo argomento sembrerebbe mostrare che i gravitoni hanno elicità -2, -1, +1 e +2, mentre in realtà hanno solo -2 e +2. Poiché l'argomento sembra essere errato o incompleto nel caso dei gravitoni, ho dei dubbi al riguardo nel caso dei fotoni. Può essere mappato in qualche modo sul trattamento standard in termini di piccolo gruppo, come descritto nella risposta di Arnold Neumaier a questa domanda http://physics.stackexchange.com/q/29766/4552?
Sembra una risposta ragionevole, ma sono completamente perso nell'affermazione che per lo spin 1 uno stato $ S_z = 0 $ sarebbe simmetrico rispetto a.rotazioni intorno ad es.l'asse $ x $.Non è certamente quello che accade per le particelle massicce;normalmente, un tale stato sarebbe caratterizzato da una simmetria completa (cioè ritorna a se stesso, e non a una versione sfasata di se stesso come fanno le polarizzazioni circolari) su una rotazione attorno all'asse di propagazione.Non dubito che la (non) esistenza di un telaio di riposo possa essere vista entrare, ma non sembra.
Non capisco questo argomento: sembra implicare che per una particella di spin-1 massiccia, puoi solo avere $ S_z = 0 $ nel frame di riposo.Ma non è corretto: l'affermazione corretta è che puoi avere $ S_z = 0 $ solo se * esiste * un frame di riposo.
@BenCrowell Penso che l'argomento di Rennie si applichi anche al gravitone.Per una particella priva di massa, la direzione della quantità di moto rompe la simmetria rotazionale (fino al piccolo gruppo).Ora sopravvive solo la rotazione attorno a questo asse.Pertanto l'elicità può essere solo ± 2.
twistor59
2012-12-12 15:04:04 UTC
view on stackexchange narkive permalink

Le risposte di KDN e John Rennie sono corrette: cercherò solo di illustrare come funziona:

I componenti di un campo di rotazione 1 senza massa soddisfano $$ \ Box ^ 2 A _ {\ mu } (x) = 0 $$ Tradizionalmente eseguiamo l'espansione nelle variabili di quantità di moto $$ A ^ {\ mu} (x) = \ int {\ frac {1} {\ sqrt {p ^ 0}} A ^ {\ mu } ({\ bf {p}}) e ^ {- ip.x}} d ^ 3 {\ bf {p}} + \ textrm {cc} $$ Se la particella si muove nella direzione z, allora il suo momento è $$ p ^ {\ mu} = (p ^ 0, 0, 0, p ^ 3) $$ e la condizione di Lorenz $ \ partial _ {\ mu} A ^ {\ mu} = 0 $ che, sulla quantità di moto le variabili spaziali hanno il seguente aspetto $$ p _ {\ mu} A ^ {\ mu} ({\ bf {p}}) = 0 $$ ora diventa $$ p ^ 0A ^ {0} ({\ bf {p}}) -p ^ 3A ^ {0} ({\ bf {p}}) = 0 $$ e quindi vediamo che $$ A ^ {0} ({\ bf {p}}) = A ^ {3} ({ \ bf {p}}) $$ Quindi possiamo esprimere $ A ^ {\ mu} ({\ bf {p}}) $ in termini di vettori di polarizzazione $$ A ^ {\ mu} ({\ bf {p} }) = \ sum \ limits _ {\ lambda} a _ {\ lambda} ({\ bf {p}}) \ epsilon ^ {\ mu} _ {\ lambda} $$ dove i tre vettori di polarizzazione sembrano $$ \ epsilon ^ {\ mu} _ {1} = (0, 1, 0, 0) $$ $$ \ epsilon ^ {\ mu} _2 = (0, 0, 1, 0) $$ $$ \ epsilon ^ {\ mu} _ {3} = (1, 0, 0, 1) $$ Se ora prendi il caso speciale di un'onda con solo la terza polarizzazione $$ A ^ {\ mu} (x) = \ int {\ frac {1} {\ sqrt {p ^ 0}} a_ {3} ({\ bf {p}}) \ epsilon_3 ^ {\ mu} e ^ {- ip.x}} d ^ 3 {\ bf {p}} + \ textrm {cc} $$ e ora calcoli $ { \ bf {E}} $ e $ {\ bf {B}} $, quindi la forma speciale di $ \ epsilon_3 ^ {\ mu} $ ti assicura di ottenere zero. Quindi la polarizzazione nella direzione della propagazione non fa nulla per contribuire al campo.

KDN
2012-12-12 11:42:58 UTC
view on stackexchange narkive permalink

L'assenza della proiezione di spin $ S_z = 0 $ è correlata all'assenza di massa del fotone. Poiché il fotone è privo di massa, si propaga alla velocità della luce e non ha evoluzione nel tempo del frame di riposo. Ciò rimuove uno degli stati di polarizzazione consentiti che sarebbero presenti per i bosoni massivi. La risoluzione del problema degli autovalori per l'operatore di spin S fornisce gli autovalori di $ S_z = \ pm \ hbar, 0 $ , dove gli autovettori normalizzati, dati in $ (x, y, z) $ notazione cartesiana, corrisponde agli autovettori $ \ frac {1} {\ sqrt {2}} (1, i , 0) $ (per $ + \ hbar $ ), $ \ frac {1} {\ sqrt {2}} (1, -i, 0) $ (per $ - \ hbar $ ) e (0,0,1) (per 0 ).
I primi due autovettori rappresentano la propagazione di fotoni polarizzati circolarmente a sinistra e a destra, rispettivamente. Il terzo autovettore rappresenta un campo non propagante. Il fotone che non si propaga, essendo privo di massa, non ha affatto energia.

La nozione di $ S_z = 0 $ fotoni virtuali tuttavia.

Grazie per la risposta @KDN ma questo è in realtà il tipo di risposta con cui mi sono sentito frustrato. Quello che mi interessa è la tua osservazione che, poiché non c'è cornice di riposo, rimuove una delle polarizzazioni consentite. Perché è esattamente questo? Inoltre, non mi è chiaro perché $ S_z = 0 $ rappresenti un campo non propagante e non semplicemente un campo propagante senza momento angolare. Se possibile, sarebbe apprezzato un chiarimento. Grazie.
@juanrga fornisce una risposta molto completa sul perché questo stato di polarizzazione svanisce. Senza la fissazione del calibro, a prima vista sembrano possibili più stati di polarizzazione, ma questi rappresentano semplicemente gradi di libertà spuri. Questi stati di polarizzazione non propaganti esistono, in senso matematico, ma non sono osservabili (anche in senso matematico, cioè non si può costruire una quantità osservabile per questi stati). L'articolo di Wikipedia sul formalismo Gupta-Bleuler (http://en.wikipedia.org/wiki/Gupta%E2%80%93Bleuler_formalism) fa un buon lavoro nell'affrontare questo problema in termini non troppo complessi.
Diracology
2016-07-01 19:14:47 UTC
view on stackexchange narkive permalink

Nella teoria quantistica dei campi gli stati di una particella sono definiti come gli stati di una rappresentazione unitaria irriducibile del gruppo di Poincaré. Se questo non fosse vero, ci sarebbero stati di una rappresentazione riducibile che non sarebbero collegati da una trasformazione di Poincaré. Questi stati sono particelle piuttosto diverse.

I Casimir

Se abbiamo una rappresentazione irriducibile di un gruppo, il Lemma di Schur dice che un operatore che commuta con tutti i generatori, un operatore Casimir, deve essere un multiplo dell'identità. Quindi l'applicazione di questo operatore a qualsiasi stato della rappresentazione fornisce lo stesso autovalore (a volte chiamato anche Casimir). Usiamo gli autovalori di diverse rappresentazioni per etichettarli. Questo è esattamente quello che facciamo in Meccanica Quantistica quando usiamo il Casimir $ J ^ 2 $ e gli autovalori $ j $ per etichettare rappresentazioni irriducibili dell'algebra del momenum angolare.

Il gruppo Poincaré ha due operatori Casimir, $ P_ \ mu P ^ \ mu $ e $ W_ \ mu W ^ \ mu $, dove $ P ^ \ mu $ è il generatore di quantità di moto e $$ W ^ \ mu = - \ frac {1} {2} \ epsilon ^ {\ mu \ nu \ sigma \ rho} J _ {\ nu \ sigma} P_ \ rho, $$ è il vettore Pauli-Lubanski. $ J ^ {\ mu \ nu} $ sono il generatore di Lorentz Group. Possiamo quindi presumere di avere due etichette per le rappresentazioni irriducibili del gruppo di Poincaré.

Scriviamo gli stati di una particella come $$ | p, \ sigma \ rangle, $$ dove $ p $ è il quattro momentum e $ \ sigma $ è l'altra etichetta da determinare. Gli autovalori di $ P_ \ mu P ^ \ mu $ sono $ m ^ 2 $, la massa quadrata della particella. Ciò dà origine a una rappresentazione dimensionale infinita i cui stati sono etichettati da quattro quantità di moto $ p $. Quindi non ci resta che trovare le rappresentazioni irriducibili dell'omogeneo Gruppo di Lorentz. Tuttavia dobbiamo considerare separatamente i casi massivi e senza massa.

Il piccolo gruppo

Prendiamo prima un particolare quattro momentum $ k $. Scriviamo una trasformazione generale del gruppo di Lorentz come $$ \ Lambda = L (\ Lambda p) W (\ Lambda, p) L ^ {- 1} (p), $$ dove $ L (p) $ è l'aumento relativo a $ k $ e $ p $, $$ L (p) k = p, $$ $$ W (\ Lambda, p) \ equiv L ^ {- 1} (\ Lambda p) \ Lambda L (p), $$ è la cosiddetta rotazione di Wigner e $ L ^ {- 1} $ denota la trasformazione inversa. Questi elementi formano il cosiddetto Piccolo gruppo che lascia il momento del frame di riposo $ k $ invariante, $$ W (\ Lambda, p) k = k. $$ Agire con $ \ Lambda $ su uno stato $ | p, \ sigma \ rangle $, $$ \ Lambda | p, \ sigma \ rangle = L (\ Lambda p) W (\ Lambda, p) | k, \ sigma \ rangle, $$ e notando che lo stato risultante deve avere quattro quantità di moto $ \ Lambda p $ ed essere in una combinazione lineare di stati con l'etichetta sconosciuta $ \ sigma $ concludiamo che $ W (\ Lambda, p) $ agisce sull'etichetta sconosciuta $ \ sigma $. Quindi conoscere la rappresentazione irriducibile del Piccolo Gruppo è ciò di cui abbiamo bisogno per conoscere le rappresentazioni irriducibili del Gruppo di Poincaré.

Particelle massicce

In questo caso possiamo andare al resto del frame, $ p ^ \ mu = (m, 0,0,0) \ equiv k ^ \ mu $. Vediamo che il piccolo gruppo che lascia $ k ^ \ mu = (m, 0,0,0) $ può essere il gruppo di rotazione in tre dimensioni, $ SO (3) $, o anche il più generale $ SU (2) $ che è una doppia copertina di $ SO (3) $. Per l'ultimo caso sappiamo (Meccanica Quantistica standard) che le loro rappresentazioni irriducibili sono etichettate dallo spin $ j = 0,1 / 2,1,3 / 2, ... $ e il numero totale di stati per un dato spin è $ 2j + 1 $.

Particelle senza massa

Non ci sono frame di pausa, quindi scegliamo $ P ^ \ mu = (k, 0,0, k) $. Il piccolo gruppo che lascia $ k $ invariante è il gruppo euclideo in due dimensioni $ ISO (2) $ che consiste di due traslazioni e rotazioni nel piano $ x ^ 1x ^ 2 $. I due generatori di traduzione danno origine a un altro valore eingen continuo $ \ theta $ ma è un fatto sperimentale che non ci sia particella con $ \ theta \ neq 0 $. Quindi dobbiamo solo considerare le rotazioni del piano. Queste rotazioni (attorno all'asse $ x ^ 3 $) formano il gruppo abeliano $ SO (2) $ i cui elementi sono $ e ^ {i \ phi \ vec J \ cdot \ vec e_3} $. Ogni rappresentazione di questo gruppo ha un solo stato e sono etichettati da numeri interi $$ h \ equiv \ vec J \ cdot \ vec e_3, $$ che chiameremo elicità.Una particella priva di massa in linea di principio ha un possibile valore dell'elicità $ h $ ma dalla sua definizione l'elicità è uno pseudo-scalare.Per una particella priva di massa che interagisce attraverso un'interazione che conserva la parità dobbiamo assegnare le due rappresentazioni $ h $ e $ -h $ per rappresentare la particella.Questo è il motivo per cui lo ftotone ha elicità $ + 1 $ e $ -1 $ e il gravitone ha elicità $ + 2 $ e $ -2 $.

Circa l'ultima equazione che hai scritto "sono etichettati da numeri interi ... che chiameremo elicità".ma penso che il problema principale sia perché questo "numero intero" non può essere zero?(corrispondente a banale ma chi ha permesso la rappresentazione)
juanrga
2012-12-12 19:03:22 UTC
view on stackexchange narkive permalink

Applicando uno schema di quantizzazione covariante al campo elettromagnetico libero $ A ^ {\ mu} $ si può mostrare l'esistenza di stati di un fotone descritti dalla quantità di moto $ k $ e una delle quattro possibili polarizzazioni stati. Questi quattro stati di polarizzazione corrispondono ai quattro possibili valori di spin -1,0,0, + 1. Quelli corrispondono ai fotoni trasversali (2), longitudinali (1) e scalari (1).

Tuttavia, ciò si ottiene assumendo che i quattro stati siano veramente indipendenti, quando non lo sono. Imponendo la condizione di Lorentz (o qualche altro equivalente come la condizione di Gupta Bleuler) si ottiene che i fotoni longitudinali e scalari sono linearmente dipendenti per ogni valore di quantità di moto

$$ [a_3 (k) - a_0 (k)] | \ Psi \ rangle = 0 $$

Qui $ a_0 $ e $ a_3 $ sono operatori di distruzione rispettivamente per fotoni scalari e longitudinali. È facile mostrare che la combinazione di cui sopra implica che i fotoni longitudinali e scalari non contribuiscono alle osservabili sul campo. Quindi il valore atteso per l'energia del campo elettromagnetico coinvolge solo fotoni trasversali

$$ \ langle \ Psi | H | \ Psi \ rangle = \ langle \ Psi | \ sum_k \ sum_ {r = 1} ^ 2 \ hbar \ omega_k a_r ^ \ dagger (k) a_r (k)] | \ Psi \ rangle $$

Di conseguenza, solo i fotoni trasversali possono essere osservate come particelle libere associate al campo elettromagnetico.

Tuttavia, i fotoni scalari e longitudinali giocano un ruolo importante in presenza di cariche . A mio parere il modo più semplice e diretto per capire il perché è utilizzare il propagatore di fotoni $ D ^ {\ mu \ nu} (k) $. Anche in questo caso dipende da quattro stati di polarizzazione. L'interpretazione del contributo del fotone trasversale $ D_T ^ {\ mu \ nu} (k) $ è diretta, mentre i contributi di longitudinale e scalare non possono essere interpretati fisicamente da separati. Tuttavia, possono essere riorganizzati in combinazioni lineari $ D_C ^ {\ mu \ nu} (k) $ e $ D_R ^ {\ mu \ nu} (k) $ che consentono una semplice interpretazione fisica

$ $ D ^ {\ mu \ nu} (k) = D_T ^ {\ mu \ nu} (k) + D_C ^ {\ mu \ nu} (k) + D_R ^ {\ mu \ nu} (k) $$

Il primo termine è il consueto contributo della radiazione e coinvolge i fotoni trasversali. Il secondo termine è il solito termine di Coulomb e coinvolge una miscela di fotoni scalari e longitudinali. Il termine rimanente, che coinvolge anche una miscela di fotoni scalari e longitudinali, non è osservabile (si può dimostrare che il suo contributo allo scattering è zero).

Si noti che sebbene l'interazione di Coulomb emerga come uno scambio di scalari e fotoni longitudinali, quei fotoni non sono osservabili. Non compaiono negli stati iniziale e finale dei processi di scattering (lo fanno solo i fotoni trasversali), ma sono particelle virtuali negli stati intermedi.

Arnold Neumaier
2016-07-01 15:26:38 UTC
view on stackexchange narkive permalink

Secondo l'elettrodinamica quantistica, quella verificata più accuratamente teoria in fisica, un fotone è un'eccitazione a particella singola del campo elettromagnetico quantistico libero. Più formalmente, è uno stato di il campo elettromagnetico libero che è un autostato del fotone operatore numero con autovalore 1.

Lo spazio di Hilbert a particella singola del fotone trasporta una rappresentazione unitaria irriducibile di spin 1 senza massa del gruppo esteso di Poincaré. Nel caso senza massa, la rappresentazione vettoriale (che è una rappresentazione di spin 1 irriducibile nel caso massiccio) è riducibile e si scompone in una rappresentazione scalare irriducibile sui modi longitudinali e in una rappresentazione irriducibile sui modi trasversali; quest'ultima è la rappresentazione del fotone.

Nello spazio della quantità di moto, i modi longitudinali hanno un potenziale vettoriale $ A (p) $ parallelo al 3-momento $ p $, e i modi trasversali hanno un potenziale vettoriale $ A (p) $ ortogonale a $ p $ (tipicamente diviso in due modalità di polarizzazione lineare o circolare). La mancanza di modi longitudinali nella rappresentazione irriducibile spiega la mancanza di $ S_z = 0 $ stati di fotoni che si propagano in $ z $ -direzione (cioè, con momento parallelo a $ (0,0,1) ^ T $).

Gli stati più generali del singolo fotone hanno la forma $ | A \ rangle = \ int \ frac {dp ^ 3} {2p_0} A (p) | p \ rangle $, dove $ | p \ rangle $ è uno stato di particella singola con 3-momento definito $ p $, $ p_0 = | p | $ è l'energia del fotone corrispondente divisa per $ c $, e l'ampiezza del fotone $ A (p) $ è una polarizzazione 3-vettore ortogonale a $ p $. Quindi un fotone generale è una sovrapposizione di onde monocromatiche con polarizzazioni, frequenze e direzioni arbitrarie.

L'ampiezza del fotone $ A (p) $ può essere considerata come la funzione d'onda del fotone nello spazio della quantità di moto. Poiché i fotoni non sono localizzabili (sebbene siano localizzabili approssimativamente), non esiste una funzione d'onda del fotone nello spazio delle coordinate con un'interpretazione della probabilità di essere localizzati in una posizione.

La trasformata di Fourier di $ A (p) $ è il cosiddetto segnale analitico $ A ^ {(+)} (x) $. Aggiungendo il suo complesso coniugato si ottiene un potenziale reale a 3 vettori $ A (x) $. In termini di ciò, le condizioni di massa zero e trasversalità si traducono insieme nelle equazioni di Maxwell libere scritte in forma di potenziale vettoriale. Estendendo il potenziale tridimensionale a un potenziale quadridimensionale aggiungendo un componente 0 evanescente e consentendo trasformazioni di gauge, le condizioni vengono portate nella forma quadridimensionale covariante delle equazioni di Maxwell libere nel gauge di Lorentz, $$ \ nabla \ cdot \ nabla A (x) = 0, ~~~~ \ nabla \ cdot A (x) = 0. $$ In particolare, un singolo fotone ha esattamente gli stessi gradi di libertà di un classico campo di radiazione nel vuoto.

[Aggiunto il 6 luglio] Si noti che i fotoni si accoppiano attraverso la materia solo attraverso la corrente di carica conservata $ j (x) $. Conservazione della carica significa che $ \ partial \ cdot j (x) = 0 $. Pertanto l'integrazione per parte implica che nella materia - interazione fotone $ \ int dx ~ j (x) \ cdot A (x) $, la parte longitudinale di $ A (x) $ è irrilevante in quanto il termine non cambia quando aggiunge a $ A (x) $ un termine longitudinale $ \ V (x) $ parziale con $ V $ scalare. Ciò mostra anche che i potenziali vettoriali senza massa e l'invarianza di gauge vanno di pari passo. Si noti inoltre che la parte Coulomb del campo elettromagnetico non è rappresentata da fotoni fisici. (Può essere visualizzato in termini di fotoni virtuali; questi non formano una rappresentazione causale del gruppo di Poincaré ma hanno tutti i 4 momenti possibili inclusi quelli tachionici e tutti i possibili stati di spin 1).

Koen
2015-10-19 23:36:20 UTC
view on stackexchange narkive permalink

I nomi dei fotoni "longitudinali" e "scalari" sono già sbagliati e non possono presentare fotoni longitudinali. Esistono due tipi di fotoni elettro-scalari longitudinali (in effetti, la componente del campo elettrico è longitudinale), che NON si annullano a vicenda se "imponiamo" una condizione di Coulomb al posto della condizione di Lorentz errata. "Imporre" condizioni di gauge è come pronunciare mezze verità o menzogne ​​complete, e questo appartiene alla scienza dell'UNFisica piuttosto che alla Fisica, perché si descrive che certi concetti teorici (onde di vuoto longitudinali) NON POSSONO ESISTERE in natura. Tali affermazioni non possono essere dimostrate da esperimenti (non si può dimostrare che qualcosa NON esiste), e dimostrare che concetti tramite esperimento è fisica, provare affermazioni negative per teoria non è fisica.

Tieni presente che "imporre condizioni di gauge" è puramente teorico che non ha base negli esperimenti.



Questa domanda e risposta è stata tradotta automaticamente dalla lingua inglese. Il contenuto originale è disponibile su stackexchange, che ringraziamo per la licenza cc by-sa 3.0 con cui è distribuito.
Loading...